Confused why Choice D is not the answer?
Hi, so I was stuck between Choice D and C. I went with choice D. The reason why is because th...
NicoleBL on April 19 at 09:36PM
  • June 1991 LSAT
  • SEC3
  • Q19
2
Replies
I-W Rule?
If the game is split into two segments... then does the order of I W matter if they are not in th...
ZoeH on April 15 at 09:38PM
  • June 2014 LSAT
  • SEC4
  • Q1
2
Replies
trying to understand why d is wrong
is see how a is right but I don't see how d is wrong. If in both years they mined the same amount...
Elizabeth25 on April 15 at 09:35PM
  • October 1996 LSAT
  • SEC1
  • Q24
1
Reply
Answer Choice B
Hi, To clarify, is B wrong because the "making of rubber" is out of the scope of the stimulu...
NicoleBL on April 15 at 02:15PM
  • June 1991 LSAT
  • SEC4
  • Q7
3
Replies
Why c and not e
Why c and bot e
Ellisw23 on April 9 at 10:09PM
  • June 1992 LSAT
  • SEC4
  • Q26
1
Reply
Why A and not B?
Integrity seems to come from the viewpoint of someone who that would weaken the argument
Ellisw23 on April 9 at 10:02PM
  • June 1993 LSAT
  • SEC3
  • Q18
1
Reply
Help
Could you please explain?
yckim2180 on April 8 at 09:30PM
  • October 2003 LSAT
  • SEC4
  • Q17
1
Reply
Can someone please explain this
Can someone please explain this? I am completely lost
EugeneC on April 8 at 09:29PM
  • October 2003 LSAT
  • SEC4
  • Q17
1
Reply
Please assit on how to appeal an LSAT question
How do I appeal a question? How am I reasonably supposed know they (LSAC) are talking about the s...
RC on April 8 at 02:16PM
  • June 2012 LSAT
  • SEC2
  • Q7
1
Reply
Why A?
Where is the support for this?
Nishant-Varma on April 8 at 02:36AM
  • October 2003 LSAT
  • SEC4
  • Q12
1
Reply
Why not B?
Can someone please explain?
Jimmywantstogotolawschool on April 8 at 02:34AM
  • October 2003 LSAT
  • SEC4
  • Q6
1
Reply
Answer choice D
Hello, how is choice D the correct answer?
amarachicynthia on April 8 at 02:33AM
  • October 2003 LSAT
  • SEC4
  • Q6
1
Reply
Why D instead of B?
I struggled between these two answers, eventually settling on B because the author seems to view ...
Williamg on April 4 at 12:03AM
  • June 1993 LSAT
  • SEC3
  • Q22
2
Replies
Please explain why A is incorrect.
If the former group didn't like to face unpleasant truths, wouldn't they withhold comments accord...
graceld on April 2 at 06:58PM
  • June 2018 LSAT
  • SEC2
  • Q18
1
Reply
B vs E
At first, I was going to select E. However, then, the lines stating in bringing suit confused me....
hfatima1 on March 31 at 02:15PM
  • October 2003 LSAT
  • SEC4
  • Q2
1
Reply
Why A and not D?
Can you explain how A is the main point?
Nishant-Varma on March 31 at 02:14PM
  • October 2003 LSAT
  • SEC4
  • Q1
1
Reply
Question
Please explain this question.
Steph on March 30 at 06:16PM
  • October 2003 LSAT
  • SEC3
  • Q9
1
Reply
Why is the answer A and not E?
Why is the answer A and not E? I don't understand how you can eliminate E.
capoleway@gmail.com on March 30 at 06:13PM
  • October 2003 LSAT
  • SEC3
  • Q7
1
Reply
Why E
explanation please.
hfatima1 on March 30 at 02:29PM
  • October 2003 LSAT
  • SEC1
  • Q18
1
Reply
Please explain
Please explain the question.
Steph on March 30 at 01:25PM
  • October 2003 LSAT
  • SEC1
  • Q25
1
Reply